Đến nội dung

Nguyenhuyen_AG nội dung

Có 785 mục bởi Nguyenhuyen_AG (Tìm giới hạn từ 20-04-2020)



Sắp theo                Sắp xếp  

#522689 $$(a^3+b)(b^3+c)(c^3+a)+10\leq 6(a^2+b^2+c^2)$$

Đã gửi bởi Nguyenhuyen_AG on 04-09-2014 - 08:47 trong Bất đẳng thức - Cực trị

Bài toán .
Ch0 các số thực dương $a,b,c$ có tổng bằng 3.Chứng minh rằng:
$$(a^3+b)(b^3+c)(c^3+a)+10\leq 6(a^2+b^2+c^2)$$

Bất đẳng thức này không đúng.




#520782 $\frac{1}{a+b}+\frac{1}{b+c...

Đã gửi bởi Nguyenhuyen_AG on 22-08-2014 - 21:01 trong Bất đẳng thức - Cực trị

Đoạn $$\sum \frac{a}{b+c}\geq \frac{(\sum a)^2}{2\sum ab}$$

Thì dấu $=$ phải ở $a=b=c$ chứ anh nhỉ ?

Em xem kỹ lại, đánh giá này có 2 dấu bằng.




#519336 CMR:$\frac{ab}{3a^2+b^2}+\frac{bc...

Đã gửi bởi Nguyenhuyen_AG on 13-08-2014 - 19:47 trong Bất đẳng thức - Cực trị

Gợi ý: Tách $3a^2+b^2=2a^2+(a^2+b^2)$ rồi sử dụng bất đẳng thức AM-GM.

 

Bạn hãy đặt bút suy nghĩ thêm tí nữa, chưa gì đã bảo không được rồi.




#519314 CMR:$\frac{ab}{3a^2+b^2}+\frac{bc...

Đã gửi bởi Nguyenhuyen_AG on 13-08-2014 - 17:49 trong Bất đẳng thức - Cực trị

Gợi ý: Tách $3a^2+b^2=2a^2+(a^2+b^2)$ rồi sử dụng bất đẳng thức AM-GM.




#519313 $\frac{1}{a+b}+\frac{1}{b+c...

Đã gửi bởi Nguyenhuyen_AG on 13-08-2014 - 17:47 trong Bất đẳng thức - Cực trị

Cho $a,b,c\geq 0$ thỏa mãn $ab+bc+ca=1$. CMR:
$$\frac{1}{a+b}+\frac{1}{b+c}+\frac{1}{c+a}-\frac{1}{a+b+c}\geq 2$$

Bất đẳng thức cần chứng minh tương đương với

$$\frac{1}{a+b}+\frac{1}{b+c}+\frac{1}{c+a}\geq 2+\frac{1}{a+b+c},$$

$$\frac{a+b+c}{a+b}+\frac{a+b+c}{b+c}+\frac{a+b+c}{c+a}\geq 2(a+b+c)+1,$$

$$\frac{c}{a+b}+\frac{a}{b+c}+\frac{b}{c+a}+2\geq 2(a+b+c).$$

Áp dụng bất đẳng thức Cauchy-Schwarz và AM-GM ta có

$$\frac{c}{a+b}+\frac{a}{b+c}+\frac{b}{c+a}+2\ge \frac{(a+b+c)^2}{2(ab+bc+ca)}+2=\frac{(a+b+c)^2}{2}+2 \geq 2(a+b+c).$$

Đẳng thức xảy ra khi $a=b,c=0$ cùng các hoán vị.




#508579 $\frac{a+b+c}{3}-\sqrt[3]{abc}...

Đã gửi bởi Nguyenhuyen_AG on 23-06-2014 - 14:15 trong Bất đẳng thức - Cực trị

Cho a,b,c là các số thực dương .CMR : 

 

      $$\frac{a+b+c}{3}-\sqrt[3]{abc}\leq max\left \{ (\sqrt{a}-\sqrt{b})^2,(\sqrt{b}-\sqrt{c})^2,(\sqrt{c}-\sqrt{a})^2 \right.\left. \right \}$$

Hãy tìm hằng số $k$ tốt nhất của bất đẳng thức

\[\frac{a+b+c}{3}-\sqrt[3]{abc}\leq k\cdot\max\left \{ (\sqrt{a}-\sqrt{b})^2,\,(\sqrt{b}-\sqrt{c})^2,\,(\sqrt{c}-\sqrt{a})^2\right\},\]

với $a,\,b,\,c$ là các số thực không âm.




#489199 Bạn đã tìm lời giải như thế nào ?

Đã gửi bởi Nguyenhuyen_AG on 28-03-2014 - 16:19 trong Tài liệu, chuyên đề, phương pháp về Bất đẳng thức

Mọi người tiếp tục nhé. Đây là bài thi quốc gia năm 2008.

 

$\boxed{\text{Bài 7.}}$ Với $x,\,y,\,z$ là ba số thực không âm đôi một khác nhau, hãy chứng minh

\[(xy+yz+zx)\left [\frac{1}{(x-y)^2}+\frac{1}{(y-z)^2}+\frac{1}{(z-x)^2} \right ] \ge 4.\]




#485266 Bạn đã tìm lời giải như thế nào ?

Đã gửi bởi Nguyenhuyen_AG on 01-03-2014 - 11:52 trong Tài liệu, chuyên đề, phương pháp về Bất đẳng thức


 Tương tự ta có bài toán sau:
Cho $a,b,c$ thực dương thỏa $2a+2b+c^2=14$. Tìm giá trị nhỏ nhất của $$P=a^3+2b^2+2c^3$$ (Trích đề thi thử Moon.vn lần 2-2014)
 

Spoiler

 

Hoàn toàn tương tự. Ta giả sử $a=x,\;b=y,\;c=z$ và sử dụng bất đẳng thức AM-GM để tạo ra $a,\;b$ và $b^2.$ Ta có

\[a^3+2b^2+2c^3 \ge \left ( 3ax^2+4by+3c^2z \right )-\left ( 2x^3+2y^2+z^3 \right ).\]

Để sử dụng được điều kiện thì ta cần tách

\[3ax^2+4by+3c^2z=2a\cdot \frac{3x^2}{2}+2b\cdot2y+c^2\cdot3z.\]

Như vậy ta cần chọn $x,\;y,\;z$ thoả mãn điều kiện

\[\left\{\begin{matrix}&\dfrac{3x^2}{2}=2y=3z \\& 2x+2y+z^2=14\end{matrix}\right.\] 

Giải hệ này ta được $x=2,\;y=3,\;z=2$ và có được $P_{\min} = 34.$




#485124 Bạn đã tìm lời giải như thế nào ?

Đã gửi bởi Nguyenhuyen_AG on 28-02-2014 - 15:14 trong Tài liệu, chuyên đề, phương pháp về Bất đẳng thức

Tình cờ thấy được bài này trên diễn đàn mọi người cùng phân tích thử xem nhé. 

 

$\boxed{\text{Bài 6.}}$ Cho hai số dương $a,\;b$ thỏa mãn điều kiện $a^2+b^2=5.$ Hãy tìm giá trị nhỏ nhất của biểu thức

\[P=a^3+b^6.\]

 

Bài này có ý tưởng AM-GM rất rõ ràng, giả thiết của bài toán chứa biểu thức có bậc nhỏ hơn bậc của $P,$ và việc sử dụng bất đẳng thức AM-GM để hạ bậc là một ý tưởng rất tự nhiên. Tuy nhiên, chúng ta lại không biết là dấu bằng của bài toán sẽ xảy ra khi nào nên việc chọn hệ số để sử dụng bất đẳng thức AM-GM cũng khá vất vả, vì vậy ta sẽ sử dụng kỹ thuật chọn dấu bằng giả định.

 

Cụ thể, ta sẽ giả sử đẳng thức của bài toán sẽ đạt được tại $a=x,\;b=y$ với $x,\;y$ là những hằng số. Khi đó theo bất đẳng thức AM-GM, ta có

\[a^3+a^3+x^3 \ge 3\sqrt[3]{a^3\cdot a^3 \cdot x^3}=3a^2\cdot x, \quad (1)\]

tức là

\[2a^3+x^3 \ge 3a^2\cdot x,\]

\[a^3\ge \frac{3a^2\cdot x-x^3 }{2}.\]

Hoàn toàn tườn tự, ta có 

\[b^6\ge 3b^2\cdot y^4-2y^6\]

Như vậy, ta có 

\[ a^3+b^6 \ge 3b^2\cdot y^4-2y^6+\frac{3a^2\cdot x}{2}-\frac{x^3}{2}=3\left ( b^2\cdot y^2+a^2\cdot \frac{x}{2} \right )-2y^6-\frac{x^3}{2}. \quad (2)\]

Ta cần chọn $x,\;y$ sao cho trong $(2)$ ta có thể sử sụng được giả thiết $a^2+b^2=5$ đồng thời cũng thoả mãn điều kiện của bait tóan, tức $x,\;y$ phải thoả mãn hệ  phương trình

\[\left\{\begin{matrix}x^2+y^2&=5 \\y^2&=\frac{x}{2}\end{matrix}\right.\]

Giải hệ này ta được $x=2,\;y=1$ và suy ra $P_{\min} =9.$

 

Nhận xét: Có một số bạn sẽ đặt câu hỏi rằng tại sao lại dùng $2$ đại lượng $a^3$ kết hợp với $x^3$ để sử dụng bất đẳng thức AM-GM cho ba số. Để ý rằng điều kiện của bài toán cho ta $a^2+b^2=5$ nên chúng ta phải cố gắng đánh giá $a^3$ sao cho xuất hiện được $a^2$ và $b^6$ sao cho xuất hiện được $b^2$ để áp dụng giả thiết $a^2+b^2=5.$ Điều này lý giải cho việc áp dụng bất đẳng thức AM-GM như trên.




#483805 Bạn đã tìm lời giải như thế nào ?

Đã gửi bởi Nguyenhuyen_AG on 18-02-2014 - 11:52 trong Tài liệu, chuyên đề, phương pháp về Bất đẳng thức

Tình cờ thấy được bài này trên diễn đàn mọi người cùng phân tích thử xem nhé. 

 

$\boxed{\text{Bài 6.}}$ Cho hai số dương $a,\;b$ thỏa mãn điều kiện $a^2+b^2=5.$ Hãy tìm giá trị nhỏ nhất của biểu thức

\[P=a^3+b^6.\]